SlideShare a Scribd company logo
1 of 107
Erectile dysfunction (ED)
By: Somayyeh Nasiripour,Pharm.D
Board of clinical pharmacy
Assistant professor at IUMS
PHYSIOLOGY OF MALE SEXUAL
FUNCTION
 Normal male sexual function requires interactions
among vascular, neurologic, hormonal, and
psychological systems.

 The initial obligatory event required for male sexual
activity, is a vascular phenomenon,
 triggered by neurologic signals and facilitated only in
the presence of an appropriate hormonal milieu and
psychological mindset.
Psychogenic erections
 triggered by neural impulses
 may originate in response to erotic visual or auditory
stimuli or be generated via fantasy.

 The centrally perceived sensual input is relayed by
neural signals to a spinal cord neural center located
at T-11 to L-2 (the thoracolumbar erection center).
 From there, neural impulses flow to the pelvic
vascular bed, redirecting blood into the corpora
cavernosae
Reflex erections
 created by tactile stimulus to the penis or genital
area which activates a reflex arc with sacral roots
originating at S-2 to S-4 (the sacral erection center).
 Psychogenic erections are more common during
man’s early sexually active years, whereas reflex
erectile activity dominates during his mature years
Nonsexual, nocturnal erections
 occurring three to four times nightly, start in early
adolescence.
 Nocturnal erectile activity may go unnoticed by sleeping
men, although most men will be aware of an erection
when they arise in the morning.
 These early morning erections often fade after urination,
creating the incorrect impression that they are a reflex
response to a full bladder.
 Nocturnal erections occur only during rapid eye
movement (REM) sleep
 depressed men rarely experience REM sleep and do not
have nocturnal or early morning erections.
 Nocturnal erections persist throughout life, although, for
as yet unexplained reasons, nocturnal erectile activity is
not as tightly coupled to REM sleep in older men
Role of
blood flow
& NO
 Normal erections require blood to flow from the hypogastric arterial
system to the paired corpora cavernosae
 As blood flow accelerates, the pressure within the intracavernosal
spaces increases dramatically, preventing penile venous outflow
from emissary veins.
 This combination of increased intracavernosal blood flow and
reduced venous outflow allows a man maintain a firm erection.
 High levels of intrapenile nitric oxide act as a local neurotransmitter
to facilitate the relaxation of intracavernosal trabeculae, thereby
maximizing blood flow and penile engorgement
 NO formed under the influence of the enzyme NO synthase, which,
in conjunction with NADPH and oxygen, transforms the substrate
amino acid arginine to citrulline and nitric oxide.
 The absolute prerequisites for penile erectile activity
are an adequate arterial inflow to provide a constant
source of intracavernosal oxygen and sufficient nitric
oxide synthase to generate nitric oxide.
 Nitric oxide acts by promoting the generation of
cyclic GMP.
 (loss of erection) occurs when nitric oxide-induced
vasodilation disappears because of metabolism of
cyclic GMP, which is primarily mediated by
intracavernosal type 5 cyclic GMP
 The role of nitric oxide may have important
therapeutic implications for patients with erectile
dysfunction (ED).

 Low intracavernosal nitric oxide synthase levels are
found in cigarette smokers and patients with
diabetes and testosterone deficiency, which may
explain why these factors are associated with a high
frequency of ED.
 sildenafil , vardenafil and tadalafil , are (PDE-5)
inhibitors.
 Interference with oxygen delivery or nitric oxide
synthesis can prevent intracavernosal blood
pressure from rising to a level sufficient to impede
venous outflow, leading to an inability to acquire or
sustain a rigid erection.
 Examples include decreased blood flow and
inadequate intracavernosal oxygen levels when
atherosclerosis involves the hypogastric artery or
other feeder vessels
 diabetes mellitus, which is associated with
suboptimal nitric oxide synthase activity
Does ED related to
hormonal deficiency
Hormonal influences
 Testosterone plays an integral role in normal male
sexual function.
 The onset of adolescent nocturnal erections
coincides with the pulsatile release of (GnRH) (LH)
activation of Leydig cell testosterone
 Testosterone deficiency results in impotence in
experimental animals and men,
 sexual potency returns when testosterone levels are
normalized
• Testosterone acts through psychogenic
channels to enhance libido.
• Testosterone is necessary for
maintenance of intrapenile nitric oxide
synthase levels
Can Normal sexual activity be
related to aging ?????
Normal sexual activity with age
 Sexual activity is affected by age, health status, and gender.
 In a population-based survey, men were more likely than women to
be sexually active and report a good quality sex life .
 Gender differences increased with age and were most noticeable in
the 75 to 85 year old group: 39 percent of men versus 17 percent of
women were sexually active.
 Men and women in good health were more likely to be sexually
active compared to those in fair or poor health.
 Men lost more years of sexually active life as a result of poor health
than women.
 Although men may remain sexually active, there are a number of
age-associated changes in sexual function in men including delay in
erection, diminished intensity and duration of orgasm, and decreased
force of seminal emission
 nine-year longitudinal follow-up study confirmed the age-associated
declines in most domains of sexual function: sexual intercourse,
erection frequency, sexual desire, satisfaction with sex, and orgasm
SEXUAL
DYSFUNCTION
erectile
dysfunction,
diminished
libido,
abnormal
ejaculation.
Decreased libido
 prevalence of reduced libido 5 to 15 % in men.
 It increases with age
 frequently accompanies other sexual disorders.
 Men with ED may experience loss of libido as a
secondary consequence of ED. This usually is
ascertained from a detailed sexual history, including
the chronology of the disorder.
 However, most patients who complain of erectile
dysfunction (ED) do not complain of reduced libido
or sexual desire.
causes of low libido include
• Medications (SSRIs, anti-androgens, 5-alpha
reductase inhibitors, opioid analgesics)
• Alcoholism
• Depression
• Fatigue
• Hypoactive sexual disorder
• Recreational drugs
• Relationship problems
• Other sexual dysfunction (fear of humiliation)
• Sexual aversion disorder
• Systemic illness
• Testosterone deficiency
 Most of these conditions are potentially treatable, so
it is important to take a good medical history, perform
a careful examination, and obtain relevant laboratory
studies to determine if any of them are present.
Sexual dysfunction associated with selective
serotonin reuptake inhibitor (SSRI) antidepressants
 reported to reduce libido in women and men, to cause
anorgasmia in women, and to increase ejaculation latency in
men
 Women are treated with SSRI medications more frequently
than men (in a ratio of 2:1) and are therefore more likely to be
affected by SSRI-related sexual dysfunction
 Estimates of the incidence of sexual dysfunction with
fluoxetine have been higher than initially thought and may be
50 percent or more
 The incidence of adverse effects on sexual function with other
SSRIs is not clear, although all SSRIs have the propensity to
cause sexual side effects
 Of note, many other antidepressants (MAOIs), TCA ,
venlafaxine and duloxetine have been associated with sexual
side effects, although this phenomenon has been less well
studied than with SSRIs.
MANAGEMENT OF SEXUAL DYSFUNCTION
•Decrease the dose
•Switch to another
SSRI
•Switch to a non-
SSRI
•Use a second drug
to offset the adverse
effects
•Drug holidays
Switch to a non-SSRI
 Bupropion , nefazodone , and mirtazapine appear to
have no or only a limited effect on sexual function
 Switching to duloxetine , released in the United
States in 2004, may also be an option.
 rates of sexual side effects in placebo-controlled
trials of duloxetine appear to be relatively low,
particularly in women
Use of a second drug
Phosphodiesterase-5 (PDE-5) inhibitors
Bupropion
Buspirone
cyproheptadine
stimulants
amantadine
yohimbine
(PDE-5) inhibitors
 Several studies indicate that the phosphodiesterase-5
inhibitor sildenafil helps relieve sexual dysfunction
caused by antidepressants
 Improvement of sexual functioning (including erectile
function, arousal, ejaculation, orgasm, and overall
satisfaction) occurred in more patients with sildenafil than
placebo (55 versus 4 %.
 sildenafil is not helpful in treating decreased libido
associated with SSRIs.
 Other phosphodiesterase-5 inhibitors are probably useful
as well
 A randomized trial of sildenafil 50 or 100 mg in 98 women
with major depression in remission (analogous to the
study in men described above found that sildenafil for
eight weeks, compared to placebo, significantly improved
scores on the Clinical Global Impression sexual function
scale
Bupropion
 (150 mg twice daily) demonstrated improvement in
sexual desire at four weeks, compared to placebo,
 in those who could tolerate the higher dose; there
was a 25 % drop out, due to bupropion side effects
(dry mouth, headache)
 A systematic review discussed a third randomized
study which also demonstrated improvement in
desire, though not orgasm, in women treated with an
SSRI and bupropion 150 mg compared to SSRI plus
placebo
Buspirone
 STUDEIS have different result
 effect of buspirone is reduced with time
 We have not found buspirone to be particularly
helpful in treating SSRI-associated sexual
dysfunction.
Drug holidays
Drug holidays
 Drug holidays have not been well studied in patients
with SSRI-induced sexual dysfunction.
 An open-label nonrandomized study of weekend
drug holidays found no benefit for patients taking
fluoxetine
 inconsistent results for patients taking paroxetine
and sertraline
Erectile dysfunction
consistent or recurrent inability to acquire or
sustain an erection of sufficient rigidity and
duration for sexual intercourse
Prevalence
 The frequency of sexual activity decreases with age
in both men and women, and sexual problems
become more common
 In men, the most common sexual problem is erectile
dysfunction (ED)
 The ages ranged from 20 to 75 years, and the
overall prevalence of ED was 16 percent. It was 8
percent in men 20 to 30 years of age and 37 percent
in men 70 to 75 years of age.
Risk factors
Low exersise
Obesity
Low sexual intercourse
smoking
Dyslipidemia
diabetes mellitus,
hypertension, obesity,
cardiovascular disease
medication use
Psychosocial factors
Neurologic causes (stroke, spinal cord or
back injury, multiple sclerosis, or
dementia.)
Association with cardiovascular disease
 Erectile dysfunction and cardiovascular disease share
many risk factors, and their pathophysiology is mediated
through endothelial dysfunction
 Cardiovascular disease and its risk factors increase the
risk for later
 erectile dysfunction may be an early warning sign of
future cardiovascular events
 men with ED without an obvious cause (eg, pelvic
trauma) and no symptoms of coronary or other vascular
disease should be screened for cardiovascular disease
and its associated risk factors prior to initiating therapy
for their sexual dysfunction, since there are potential
cardiac risks associated with sexual activity in patients
with heart disease
Drugs(25 % of cases of ED)
Antidepressants - Most antidepressants are associated with ED,
but in particular, the selective serotonin reuptake inhibitors (SSRIs)
Spironolactone
Sympathetic blockers such as clonidine , guanethidine, or
methyldopa
Thiazide diuretics
Ketoconazole
Cimetidine , but apparently not ranitidine or famotidine
It had been thought that beta blockers are an important cause of
ED, but a systematic review of randomized, controlled trials found
only a small increased risk of sexual dysfunction with beta blocker
therapy
Common over-the-counter products contain chemicals like nicotine
and alcohol that have a disruptive effect on male sexual function
Although some recreational drugs, such as cocaine and heroin, can
initially stimulate libido and sexual arousal, they ultimately exert a
negative impact on the ability to acquire and sustain erectile function
association of ED with bicycling
prolonged pressure on the pudendal and cavernosal nerves or
compromises blood flow to the cavernosal artery can result in penile
numbness and impotence.
The penile numbness has been attributed to the pressure on the perineal
nerves
ED is thought to be due to a decrease in oxygen pressure in the pudendal
arteries
Endocrine disorders
 Testosterone deficiency affects peripheral mechanisms
that are responsible for penile erections
 Testosterone deficient men are still capable of exhibiting
some erectile activity during nocturnal However, the
penile swelling in this setting usually is not of sufficient
rigidity to permit vaginal penetration.
 This defect is corrected after normalization of
testosterone levels, probably due to restoration of
intrapenile nitric oxide synthase levels.
 In a study of 1162 men, serum testosterone levels <225
ng/dL associated with an increased frequency of sexual
dysfunction
 he effects of testosterone on libido are more consistent
than on erectile function
 Testosterone replacement can restore nocturnal
erections in hypogonadal men but the effects are greater
when testosterone plus a PDE-5 inhibitor are
administered
 Other disruptions in hormone secretion, including
hyperprolactinemia, hyperthyroidism, and hypothyroidism
are commonly associated with ED
 recognition that 1/3 of men with type 2 DM have
subnormal testosterone concentrations suggests that this
hormone deficiency, and not just diabetic
vasculopathy/neuropathy, may play a role in the ED so
commonly seen in men with diabetes
Treatment of male sexual
dysfunction
GENERAL PRINCIPLES
 first step in the treatment of male sexual dysfunction
is to determine the etiology. Certain clues in the
history and appropriate laboratory testing can usually
establish the primary cause
Clinical clues to causes of male sexual
dysfunctio
• Psychogenic
• Genitourinary trauma - eg, radical prostatectomy
Rapid onset
• Anxiety
• Vascular steal
Nonsustained
erection
• Depression
• Drug-induced
Depression or use
of certain drugs
Complete loss of
nocturnal erections
Vascular disease
Neurologic disease
Therapy of men with erectile dysfunction is aimed at restoration
of the two vital sexual functions:
the capacity to acquire and sustain penile erections;
reactivation of libido.
Optimal treatment varies with the cause of the erectile
dysfunction
Phosphodiesterase-5 (PDE-5) inhibitors, penile self-injection
programs with vasoactive drugs, vacuum erection devices, or
penile prostheses allow many men with vasculogenic,
neurogenic, or psychogenic erectile dysfunction to acquire and
maintain erections.
For first-line therapy, we recommend the phosphodiesterase inhibitors because of their efficacy, ease
of use, and favorable side effect profile. Sildenafil , vardenafil , and tadalafil appear to be equally
effective, but tadalafil has a longer duration of action. (See 'Pharmacotherapy' below and 'Choice of
drug' below.) Phosphodiesterase inhibitors are contraindicated in men taking nitrates .
For second-line therapy, we recommend consideration of penile self-injectable drugs, intraurethral
alprostadil , and vacuum devices. (See 'Penile self-injection' below and 'Vacuum-assisted erection
devices' below.)
Surgical implantation of a penile prosthesis should be reserved for men who cannot use or who have
not responded to first- and second-line therapies. (See 'Penile prostheses' below.)
For men with sexual dysfunction and low serum testosterone levels, ie, hypogonadism, testosterone
replacement therapy should be the initial treatment. (See "Testosterone treatment of male
hypogonadism" .)
Psychotherapy alone or in combination with psychoactive drugs may be helpful in men whose erectile
dysfunction is caused by depression or anxiety. In addition, alpha adrenergic blocking agents such as
yohimbine hydrochloride have enjoyed some popularity as a remedy for men with psychogenic erectile
dysfunction. (See "Overview of male sexual dysfunction" .)
In obese men with erectile dysfunction, weight loss and increased physical activity are associated with
an improvement in erectile function in about one third of patients
Lifestyle modifications such as diet and exercise have
improved ED in studies of men with hypertension,
diabetes, or the metabolic syndrome
A meta-analysis of four randomized trials of lifestyle
modification interventions in 597 men with ED
demonstrated statistically significant improvement in
sexual function
Statins improved the response to sildenafil in two
studies of men with dyslipidemia and ED
PHARMACOTHERAPY
Phosphodiesterase 5 inhibitors
 treatment options now include three
phosphodiesterase-5 (PDE-5) inhibitors: sildenafil ,
vardenafil , and tadalafil .
 All act to increase intracavernosal cyclic GMP levels,
 and each one has been proven to be effective in
restoring erectile function, allowing for satisfactory
sexual intercourse in many men with erectile
dysfunction.
 However, PDE-5 inhibitors will not work without
sufficient environmental and psychological cues that
result in sufficient sexual arousal and stimulation to
initiate the physiological changes in the penis.
sildenafil
Sildenafil
 Sildenafil (Viagra) was the first orally administered treatment of
proven efficacy for erectile dysfunction. Taken one hour before
planned sexual intercourse, it is effective for a wide range of
disorders causing erectile dysfunction. Nevertheless, it should not be
prescribed before performing a systematic assessment of the cause
of erectile dysfunction, including a history and physical examination
and when appropriate, measurement of serum testosterone,
prolactin, and TSH.
 rationale for the use of sildenafil and other PDE-5 inhibitors is based
upon the role of nitric oxide-induced vasodilation (which is mediated
by cyclic GMP) in initiating and maintaining an erection.
 Detumescence is associated with catabolism of cyclic GMP by type 5
phosphodiesterase.
 PDE-5 inhibitors act by blocking the latter enzyme and as a result
increase both the number and duration of erections in men with
erectile dysfunction
Efficacy
 In a dose-escalation study of 25 to 100 mg of
sildenafil in 532 men with organic (70 percent),
psychogenic (11 percent), or mixed erectile
dysfunction (18 percent), 69 % of all attempts at
sexual intercourse were successful in the men who
took sildenafil compared to 22 percent of those who
took placebo
 All do not respond to the drug
 56 % of diabetic men have improved erections with
sildenafil
 In men with prostate cancer who have undergone
prostatectomy or radiation therapy, sildenafil appears
to be effective in approximately 50 % of patients
addition of exogenous
testosterone to sildenafil
addition of exogenous testosterone to sildenafil therapy may be useful in men with
serum total testosterone concentrations <400 ng/dL who do not respond to
sildenafil alone.
This was illustrated in a study of 75 such patients who were randomly assigned to
receive daily testosterone gel (5 gm/day) or placebo in addition to sildenafil
A greater improvement in erectile function (as measured by the IIEF) was observed
in the testosterone-sildenafil group when compared with placebo.
Although serum total and free testosterone concentrations increased in the
testosterone-sildenafil group, no correlation was seen between testosterone levels
and erectile function.
The clinical implications of these findings are unclear, as the lower limit of normal
for most total testosterone assays is <280 to 300 ng/dL. Therefore, many of the
men in this study were not truly hypogonadal.
Dose
 For maximum effectiveness, sildenafil should be
taken orally about one hour before a planned sexual
encounter.
 The initial dose should be 50 mg,
 it should be reduced to 25 mg if side effects occur.
 If, well tolerated but the erectile response is not fully
satisfactory, the dose can be increased to 100 mg.
 The duration of action is approximately four hours
Cardiovascular effects
Sildenafil is a vasodilator that lowers the blood pressure by about 8 mmHg; this change typically produces no
symptoms
The combination of sildenafil and nitrates can lead to severe hypotension (eg, more
than 50/25 mmHg) and syncope.
As a result, sildenafil is contraindicated in patients taking nitrates of any form, regularly
or intermittently.
If a man who has taken sildenafil has an acute ischemic syndrome, nitrates should not
be prescribed within 24 hours (or longer in patients with renal or hepatic dysfunction).
Sildenafil has been associated with myocardial infarction and sudden death.
However, since sexual activity is also associated with these complications, case reports of MI in association
with sildenafil may have been unrelated to the drug.
Consistent with this hypothesis are the results of extensive postmarketing surveillance, as well as a meta-
analysis, which have failed to demonstrate a significant association between sildenafil and cardiac events
Sildenafil does not appear to have adverse effects on coronary hemodynamics, even in men with either stable or
severe coronary disease
These data support the positions of the Second Princeton Consensus Panel and a consensus statement from
the American College of Cardiology/American Heart Association that a PDE-5 inhibitor is safe for men with
stable coronary artery disease who are not taking nitrates
The vasodilator properties of sildenafil may have an adverse effect in some patients with a hypertrophic
cardiomyopathy; the decrease in preload and afterload can increase the outflow obstruction, culminating in an
unstable hemodynamic state
Interactions
 concurrent use (regularly/intermittently) of organic nitrates in any
form (eg, nitroglycerin, isosorbide dinitrate) is X interaction and
should be avoided
 Alpha adrenergic antagonists, which are commonly used for the
treatment of benign prostatic hyperplasia, may cause symptomatic
hypotension when taken in combination with PDE-5 inhibitors
 Tamsulosin is the least likely to cause orthostatic hypotension
 Current labeling for all three PDE-5 inhibitors recommends that
patients should be stable on alpha blocker therapy prior to initiating a
PDE-5 inhibitor (which should then be started at the lowest
recommended dose).
 Conversely, in patients already taking a PDE-5 inhibitor, alpha
blocker therapy should be initiated at the lowest dose.
 Sildenafil should also be avoided in patients taking drugs that can
prolong the half-life of sildenafil by blocking CYP3A4 (including
erythromycin , ketoconazole , protease inhibitors, and grapefruit
juice); if used, the potential adverse effects should be stressed with
the patient and sildenafil should be started at a dose of 25 mg rather
than the 50 mg dose.
Visual effects
 Although sildenafil acts by inhibiting type 5 phosphodiesterase, it can affect the retina
via weak inhibition of type 6 phosphodiesterase, which is required for the
transformation of light into electrical signals.
 In a brief report of five normal men, a single 100 mg dose of sildenafil caused changes
in the electroretinogram one hour later that were completely reversible at six hours
.There were no changes in visual acuity, visual fields, or visual-evoked potentials.
 Sildenafil also causes blue vision in approximately 3 percent of men. This effect lasts
two to three hours and disappears spontaneously.
 Rare cases (23 cases since 1998) of nonarteritic anterior ischemic optic neuropathy
(NAION) have been reported in men taking sildenafil and tadalafil
 Most of these cases occurred in men with underlying autonomic or vascular risk
factors.
 NAION shares a number of risk factors with erectile dysfunction: age over 50,
hypertension, dyslipidemia, and diabetes. Therefore, it is unclear whether NAION in
these patients is due to their underlying vascular risk factors, anatomic defects, or use
of sildenafil. However, the United States Food and Drug Administration has ordered
that warnings be added to labeling for sildenafil, tadalafil, and vardenafil to reflect this
potential risk
 The long-term effects of sildenafil on visual function
are unknown.
 At this time, monitoring of visual function does not
appear to be necessary in the average man without
retinal disease
 Although there are no clinical data on the safety and
efficacy of sildenafil in men with retinitis pigmentosa,
the manufacturer recommends caution in these
patients.
Hearing loss
 Sildenafil , vardenafil , and tadalafil use have been
associated with rare reports (n = 29) of sudden
hearing loss
 Although no causal relationship has been
demonstrated, the FDA has approved labeling
changes for all three drugs to include this potential
risk.
 The hearing loss was temporary in approximately
one-third of patients, and ongoing (or unknown) in
the remaining two-thirds at the time of the FDA
report.
Other side effects
 Side effects associated with sildenafil are related to
its vasodilatory properties and are similar to those
induced by nitrates.
 These include headache, lightheadedness,
dizziness, flushing, distorted vision, and, in some
cases, syncope.
Recreational use
 Because sildenafil treatment is associated with a
marked reduction in the post-ejaculatory refractory
time, men are capable of having a second erection in
a shorter time frame than was possible without this
therapy.
 As a result, recreational use of sildenafil is common.
 ,there are case reports of stroke in men taking high
dose sildenafil
Vardenafil
 Vardenafil shares a similar structure, onset and
duration of action and side-effect profile with
sildenafil ,
 available as a 10 and 20 mg dose
 Although there are no direct comparison studies,
vardenafil appears to be as effective as sildenafil
 The duration of action, like sildenafil, is
approximately four hours
 High-fat, but not moderate-fat meals, may lower
vardenafil 's peak serum concentration by
approximately 18 percent, and delay its absorption
by one hour
 A slight prolongation of the QT interval may occur,
but this is not thought to be clinically important
 However, vardenafil should not be used in men with
congenital QT prolongation or in those taking
antiarrhythmics drugs such as quinidine ,
procainamide , amiodarone , or sotalol .
Side effects
 similar to those seen with sildenafil , and include
headache, flushing, and rhinitis, in 13, 10, and 10
percent, respectively
 In one trial, side effects decreased over time
 Changes in color vision (blue vision) have not been
reported thus far
Tadalafil
 has a different chemical structure than sildenafil and
vardenafil
 Although there are no direct comparison studies,
tadalafil also appears to be as effective as sildenafil
but has a longer duration of action
 In addition, it does not cause blue vision (as
described below),
 but otherwise shares a similar side-effect profile with
the other two phosphodiesterase inhibitors.
Dose:
 The recommended starting dose for as-needed use is 10 mg, increasing to 20 mg if necessary
 Lower doses of tadalafil (2.5 mg, 5 mg) are available for once daily administration .this approach
appears to be as effective as taking higher doses on an as-needed basis
 Food does not interfere with its absorption [ 56 ].
 Current FDA labeling includes warnings about daily tadalafil use and drug interactions (the once-
daily dose should not exceed 2.5 mg in patients receiving CYP3A4 inhibitors such as ritonavir ,
ketoconazole , and itraconazole ).
 Alcohol should be avoided
 caution is advised in patients with hepatic dysfunction.
 Daily dose tadalafil should not be used in patients with severe renal insufficiency
In an integrated analysis of randomized trials in 1112 men with erectile dysfunction receiving
2.5, 5, 10, or 20 mg of tadalafil or placebo, 75 percent of intercourse attempts were successful
in the tadalafil group compared with 32 percent with placebo
In a second study in 348 men, 59 percent of intercourse attempts were successful at 36
hours with tadalafil (20 mg) compared with 28 percent with placebo [ 51 ].
Side effects
 Side effects are similar to those seen with sildenafil and
vardenafil with headache, dyspepsia, flushing, and
rhinitis occurring in 8 to 14, 5 to 10, 4 to 6, and 5 percent,
respectively
 The duration of these side effects is not known.
 In one study, back pain occurred in 6 percent of patients
taking the 20 mg dose .In general, the pain occurred 12
to 24 hours after dosing, was mild, and resolved within 48
hours.
 Visual side effects have not been described.
 A possible association with sudden hearing loss is
described above.
Drug interactions
 Like sildenafil and vardenafil , tadalafil is contraindicated in men taking
concurrent nitrates.
 In spite of its longer half life, the time course of nitrate interaction with
tadalafil does not appear to be prolonged
 However, it has been suggested that nitrates should be avoided for at least
48 hours after the last tadalafil dose
 Other issues related to sexual activity in men with coronary heart disease
are similar to those with sildenafil.
 Alpha adrenergic antagonists, which are commonly used for the treatment of
benign prostatic hyperplasia, may cause symptomatic hypotension when
taken in combination with PDE-5 inhibitors
 Tamsulosin is the least likely to cause orthostatic hypotension
 Current labeling for all three PDE-5 inhibitors recommends that patients
should be stable on alpha blocker therapy prior to initiating a PDE-5 inhibitor
(which should then be started at the lowest recommended dose).
Conversely, in patients already taking a PDE-5 inhibitor, alpha blocker
therapy should be initiated at the lowest dose.
 Excessive alcohol intake (five or more drinks) in combination with tadalafil
administration may potentiate the hypotensive effect of tadalafil
 Use of potent CYP3A4 inhibitors should be avoided as described above for
sildenafil
Vision
 Rare case reports of nonarteritic anterior ischemic optic neuropathy
have been described with the PDE-5 inhibitors, prompting the United
States Food and Drug Administration to require revised labeling to
reflect this potential risk
 The PDE-5 inhibitor in sildenafil cross-reacts with the retinal PDE-6
and accounts for the transient "blue vision" side effect reported in a
small percentage of sildenafil-treated men.
 Tadalafil does not appear to disrupt PDE-6 so blue vision has not
been reported with this medication.
 Daily use of either tadalafil or sildenafil does not appear to be
associated with retinal toxicity, as illustrated by a trial of 143 men
who were randomly assigned to receive sildenafil 50 mg, tadalafil 5
mg, or placebo .After six months of therapy, there were no
abnormalities in visual function or electroretinograph recordings
suggestive of drug toxicity in the treatment groups compared to
placebo.
Reproductive effects
 Tadalafil does show some cross reactivity with PDE-11,
an enzyme that is present in the testes and pituitary, but
the physiologic significance of this enzyme is unclear.
 Tadalafil does not appear to disrupt any aspect of normal
pituitary or testicular function
 This was illustrated in a study of 421 men over age 45
who were randomly assigned to receive placebo (n =
101) versus tadalafil 10 mg (n = 103), or placebo (n =
106) versus tadalafil 20 mg (n = 111) daily for six months.
 During this interval, when compared to placebo, six
months of daily administration of tadalafil caused no
change in sperm concentration, sperm count, motility or
morphology and did not alter serum LH, FSH or
testosterone concentrations.
Comparisons
 All PDE-5 inhibitors allow men to have erections after appropriate
sexual stimulation but differ in the onset of action as well as duration
of effectiveness.
 Tadalafil differs in two ways. Its absorption is less affected by high
fat meals and alcohol and it has a longer duration of action.
 Sildenafil has the longest safety record of the three drugs.
 With sildenafil and vardenafil men are advised that maximum
effectiveness is achieved by taking the tablet on an empty stomach
(high fat meals and alcohol appear to delay the absorption of both
drugs) and then wait at least an hour before attempting sexual
intercourse.
 Tadalafil can be taken without regard to meals.
 The period of effectiveness for a single dose of sildenafil or
vardenafil is four hours. For men with mild to moderate ED, the
efficacy of sildenafil may persist for 8 to 12 hours [
 Men with erectile dysfunction may have an erection in response to
sexual stimulation for up to 36 hours after a single tadalafil dose
 In one international multi-center double-blind crossover study
comparing patient preference for sildenafil 50 mg or tadalafil
20 mg, 215 men (mostly sildenafil naïve men) with erectile
dysfunction age 65 and younger were randomized to receive
each medication for four weeks with an intervening one week
washout period before switching to the other medication .The
following results were seen:
 Men reported a similar number of sexual intercourse
episodes: 2334 with tadalafil and 2233 with sildenafil .
 At the end of the study, 66.3 percent of men expressed a
preference for tadalafil and 33.7 percent for sildenafil as a
treatment for their erectile dysfunction.
 The interval between dosing and sexual intercourse
differed. On average sildenafil -treated men had sex 2.2 hours
after dosing well within the 4 hour window of opportunity
stipulated on the label whereas tadalafil -treated men were
able to maintain efficacy but were able to delay sexual
intercourse for 5.5 hours after dosing.
Choice of drug
 Currently available PDE-5 inhibitors result in similarly
high rates of successful sexual intercourse (68 to 69
percent compared to 33 to 35 percent for placebo),
and similar side effect profiles
 Therefore, a Clinical Practice Guideline from The
American College of Physicians recommends that
the choice of PDE-5 inhibitor be based upon on the
patient's preferences, including cost, ease of use,
and adverse effects
Supplements
containing PDE-5
inhibitors
 The FDA has issued a warning to consumers not to
purchase or consume dietary supplements that claim
to increase sexual stamina, confidence, and
performance
 Current studies suggest that one-third to one-half of
supplements claiming to be “natural” products for
sexual enhancement contain synthetic chemicals,
most commonly, PDE-5 inhibitors or analogs of PDE-
5 inhibitors
 The concern is that patients who take nitrates for
cardiovascular disease may experience a drastic
lowering of blood pressure if these supplements are
consumed.
Summary
 All three PDE-5 inhibitors: sildenafil , vardenafil and tadalafil work to sustain
levels of cyclic GMP within the penile corpora cavernosae to allow men with
erectile dysfunction to achieve erections in response to appropriate sexual
stimuli.
 Duration of action separates one PDE-5 inhibitor from another. Sildenafil and
vardenafil are effective as early as 30 minutes and up to four hours after
dosing
 whereas tadalafil is effective as early as 16 minutes after and up to 36 hours
after dosing.
 Sildenafil and vardenafil must be taken on an empty stomach, while tadalafil
can be taken without regard to food.
 An important factor in the success of PDE-5 inhibitor therapy is instruction
and counseling on proper use and administration.
 Sildenafil , vardenafil , and tadalafil result in similarly high rates of successful
sexual intercourse
 Side effect profiles are similar; only rarely do men with erectile dysfunction
discontinue treatment because of side effects. Current practice guidelines
recommend that the choice of PDE-5 inhibitor should be based upon on the
patient's preferences, including cost, ease of use, and adverse effects
Penile self-injection
Intrapenile injection therapy with alprostadil (prostaglandin E1,
Caverject), papaverine , or alprostadil with papaverine and
phentolamine (Tri-Mix) have all been used for purposes of
inducing erection.
 The sympathetic nervous system normally maintains the penis in a flaccid or
non-erect state.
 All of these vasoactive drugs, when injected into the corpora cavernosae,
inhibit or override sympathetic inhibition to encourage relaxation of the
smooth muscle trabeculae within the penile erectile bodies.
 . The combination of accelerated arterial inflow and impeded venous outflow
from the corpora cavernosae creates an erection
 Considerable education is required for men to become facile with this form of
therapy.
 Under the guidance of urologists men are trained in sterile methods and the
proper technique for inserting an insulin syringe with a 26 gauge needle
through the shaft of the penis and injecting the vasoactive agent into
one corporeal body
 The cross circulation of the penile corpora allows medication injected into
one penile corporeal body to diffuse over to the contralateral side so that a
full, firm erection can be expected within a few minutes after intrapenile
installation of the drug
 In the early days of penile self-injection therapy,
papaverine proved to be more reliable than
phentolamine in producing an erection.
 Papaverine is a parenteral vasodilator with marginal
efficacy in peripheral vascular disease,
 phentolamine is an alpha-adrenergic blocker used
for hypertensive crises in pheochromocytoma.
Alprostadil
 originally introduced to maintain the patency of the
ductus arteriosus before definitive cardiac surgery
could be undertaken.
 When it became apparent that it was effective in a
penile self-injection program, the pharmaceutical
company making alprostadil was granted approval
by FDA to market alprostadil under the name
Caverject as a treatment for erectile dysfunction.
 alprostadil and papaverine remain popular as
monotherapy, and all three drugs can be given
together.
 The efficacy and safety of a program of at-home
intrapenile alprostadil injections were then
investigated in a six-month study
 683 men with vasculogenic, neurogenic,
psychogenic, and mixed causes of erectile
dysfunction entered the trial and 471 (69 percent)
completed the trial.
 The reasons for drop out included penile pain, lack
of efficacy, loss to follow-up, dislike of self injection,
and other problems.
 satisfactory by 87 percent of the men completing the
trial and 86 percent of their partners.
Side effects
penile pain, occurring in 50 percent.
Pain was the side effect most often cited by men
who discontinued therapy.
Priapism
Papaverine injected into the corpora cavernosae
may escape into the general circulation and can
be hepatotoxic; abnormal liver function tests
have been reported in men using this drug for
penile self-injection
In contrast, alprostadil is metabolized within the
corpora and does not adversely affect the liver.
Priapism
 Priapism, or a prolonged erection lasting more than
four to six hours, is a medical emergency often
requiring immediate urologic attention to evacuate
blood clogged within the corpora cavernosae [ 75 ].
Prolonged erections occur in 6 percent of men who
use intrapenile alprostadil and about 11 percent of
those who use intrapenile papaverine .
 Lower doses should be used in men with
neuropathic erectile dysfunction (due to spinal cord
injury or multiple sclerosis), due to their risk of
priapism.
One study evaluated the effects of
prolonged priapism
 Most priapism that lasted 36 hours could be treated
successfully by puncture and alpha-adrenergic drugs
without any fibrosis of the corpora cavernosa.
 After 48 hours, glandulocavernosal shunts were
required to achieve detumescence. All the men
developed fibrosis of the corpora cavernosa and all
but one were unable to continue with the injections
of vasoactive drugs.
Intraurethral alprostadil
 Intraurethral administration of alprostadil (MUSE)
provides a less invasive alternative to intrapenile
injection.
 The efficacy of intraurethral alprostadil was evaluated in
a double-blind, placebo-controlled trial in 1511 men with
chronic erectile dysfunction from a variety of organic
causes
 2/3 of these men responded to intraurethral alprostadil
with an erection sufficient for intercourse;
 these men were then randomly assigned to therapy with
either alprostadil or placebo. Successful intercourse on at
least one occasion was much more likely with alprostadil
(65 versus 19 percent with placebo).
 Among the men who responded to alprostadil, 7 of 10
applications were followed by successful intercourse.
After insertion of the alprostadil into the urethra, the penis should be massaged
for up to one minute to ensure equal distribution in the corpora cavernosae.
Side effects
 systemic effects were uncommon,
 complications such as priapism and penile fibrosis
were less common than after alprostadil given by
penile injection.
 The 19 percent response rate in the placebo group
suggests that psychogenic factors were responsible
for the sexual dysfunction, since placebo injections
do not induce erections in solely organic causes of
impotence.
VACUUM-ASSISTED
ERECTION DEVICES
 increased arterial inflow and occlusive rings to
discourage venous egress from the penile corpora
cavernosae
 A certain amount of mechanical dexterity is required to
use these devices effectively, but once men become
comfortable with using the vacuum and restraining rings
they can create an erection sufficient for vaginal
penetration and sexual intercourse.
 They cannot, however, ejaculate externally because the
occlusive rings that prevent venous drainage also
compress the penile urethra sufficiently to prevent
seminal fluid from reaching and traversing the urethral
meatus.
 A number of devices are available for purchase over the
counter.
 Vacuum devices successfully create erections in as many
as 67 percent of patients.
 Satisfaction with vacuum-assisted erections has varied
between 25 and 49 percent.
PENILE PROSTHESES
 This form of therapy remains a viable option for
those men who do not respond to sildenafil and find
penile injection or vacuum erection therapy
distasteful
 The placement of penile implants requires surgery
 Side effects include those related to the anesthesia,
local wound infections, and mechanical failure
necessitating surgical removal and reimplantation of
a new functioning prosthesis.
Yohimbine
5.4 mg TID
 blocking presynaptic alpha-2-adrenergic receptors,
yohimbine increases cholinergic and decreases
adrenergic tone, changes that should theoretically be
effective in men with psychogenic erectile
dysfunction
 Optimal results are achieved when yohimbine use is
restricted to men with psychogenic erectile
dysfunction
 Treatment is continued only in those who respond
and also tolerate the drug's side effects, which
include dizziness, flushing, nausea, and headache.
Androgen replacement
therapy
Testosterone should be administered only to a man who is hypogonadal , as evidenced by clinical
symptoms and signs consistent with androgen deficiency and a distinctly subnormal serum
testosterone concentration.
In comparison, increasing the serum testosterone concentration in a man who has symptoms
suggestive of hypogonadism but whose testosterone concentration is already normal will not relieve
those symptoms. (See "Clinical features and diagnosis of male hypogonadism" .)
Testosterone can be replaced satisfactorily whether the testosterone deficiency is due to primary or
secondary hypogonadism.
The principal goal of testosterone therapy is to restore the serum testosterone concentration to the
normal range
The role of testosterone replacement to treat the decline in serum testosterone concentration that
occurs with increasing frequency above age 60 in the absence of identifiable pituitary or hypothalamic
disease is uncertain
erectile dysfunction

More Related Content

What's hot

Progesterone rise on the day of hcg administration (ppremature luteinization)...
Progesterone rise on the day of hcg administration (ppremature luteinization)...Progesterone rise on the day of hcg administration (ppremature luteinization)...
Progesterone rise on the day of hcg administration (ppremature luteinization)...Aboubakr Elnashar
 
Hormone replacement therapy dr. sharda jain
Hormone replacement therapy dr. sharda jainHormone replacement therapy dr. sharda jain
Hormone replacement therapy dr. sharda jainLifecare Centre
 
Role of Dydrogesterone in Recurrent Pregnancy Loss Dr Sharda Jain
Role of Dydrogesterone in Recurrent Pregnancy  Loss Dr Sharda Jain Role of Dydrogesterone in Recurrent Pregnancy  Loss Dr Sharda Jain
Role of Dydrogesterone in Recurrent Pregnancy Loss Dr Sharda Jain Lifecare Centre
 
Use of progesterone in obstetrics &amp; gynaecology namkha presents
Use of progesterone in obstetrics &amp; gynaecology namkha presentsUse of progesterone in obstetrics &amp; gynaecology namkha presents
Use of progesterone in obstetrics &amp; gynaecology namkha presentsnamkha dorji
 
Tadalista (Tadalafil Tablets)
Tadalista (Tadalafil Tablets)Tadalista (Tadalafil Tablets)
Tadalista (Tadalafil Tablets)Clearsky Pharmacy
 
Thin Endometrium & Infertility (Part – II) , Dr. Sharda Jain, Life Care Centre
Thin Endometrium & Infertility(Part – II) , Dr. Sharda Jain, Life Care Centre Thin Endometrium & Infertility(Part – II) , Dr. Sharda Jain, Life Care Centre
Thin Endometrium & Infertility (Part – II) , Dr. Sharda Jain, Life Care Centre Lifecare Centre
 
Hypogonadism and testosterone replacement
Hypogonadism and testosterone replacementHypogonadism and testosterone replacement
Hypogonadism and testosterone replacementPeninsulaEndocrine
 
Uncosummated marriage
Uncosummated marriageUncosummated marriage
Uncosummated marriagePolash Roy
 
Baiju dhea 15.11.12
Baiju dhea 15.11.12Baiju dhea 15.11.12
Baiju dhea 15.11.12Asha Reddy
 
Role of antioxidant in male infertility
Role of  antioxidant in male infertilityRole of  antioxidant in male infertility
Role of antioxidant in male infertilityLifecare Centre
 
Erectile Dysfunction
Erectile DysfunctionErectile Dysfunction
Erectile DysfunctionEko indra
 
Androgens and antiandrogens
Androgens and antiandrogensAndrogens and antiandrogens
Androgens and antiandrogensGITAM University
 
Uses of of N-acetyl Cysteine in Medicine, obstetrics, gynecology and inferti...
Uses of  of N-acetyl Cysteine in Medicine, obstetrics, gynecology and inferti...Uses of  of N-acetyl Cysteine in Medicine, obstetrics, gynecology and inferti...
Uses of of N-acetyl Cysteine in Medicine, obstetrics, gynecology and inferti...Aboubakr Elnashar
 

What's hot (20)

Progesterone rise on the day of hcg administration (ppremature luteinization)...
Progesterone rise on the day of hcg administration (ppremature luteinization)...Progesterone rise on the day of hcg administration (ppremature luteinization)...
Progesterone rise on the day of hcg administration (ppremature luteinization)...
 
Hormone replacement therapy dr. sharda jain
Hormone replacement therapy dr. sharda jainHormone replacement therapy dr. sharda jain
Hormone replacement therapy dr. sharda jain
 
Role of Dydrogesterone in Recurrent Pregnancy Loss Dr Sharda Jain
Role of Dydrogesterone in Recurrent Pregnancy  Loss Dr Sharda Jain Role of Dydrogesterone in Recurrent Pregnancy  Loss Dr Sharda Jain
Role of Dydrogesterone in Recurrent Pregnancy Loss Dr Sharda Jain
 
Dhea
DheaDhea
Dhea
 
Use of progesterone in obstetrics &amp; gynaecology namkha presents
Use of progesterone in obstetrics &amp; gynaecology namkha presentsUse of progesterone in obstetrics &amp; gynaecology namkha presents
Use of progesterone in obstetrics &amp; gynaecology namkha presents
 
Tadalista (Tadalafil Tablets)
Tadalista (Tadalafil Tablets)Tadalista (Tadalafil Tablets)
Tadalista (Tadalafil Tablets)
 
Menopausal Hormone Replacement Therapy by Dr Shahjada Selim
Menopausal Hormone Replacement Therapy by Dr Shahjada SelimMenopausal Hormone Replacement Therapy by Dr Shahjada Selim
Menopausal Hormone Replacement Therapy by Dr Shahjada Selim
 
Dhea jdr
Dhea jdrDhea jdr
Dhea jdr
 
Dhea references
Dhea referencesDhea references
Dhea references
 
Thin Endometrium & Infertility (Part – II) , Dr. Sharda Jain, Life Care Centre
Thin Endometrium & Infertility(Part – II) , Dr. Sharda Jain, Life Care Centre Thin Endometrium & Infertility(Part – II) , Dr. Sharda Jain, Life Care Centre
Thin Endometrium & Infertility (Part – II) , Dr. Sharda Jain, Life Care Centre
 
Hypogonadism and testosterone replacement
Hypogonadism and testosterone replacementHypogonadism and testosterone replacement
Hypogonadism and testosterone replacement
 
Progesterone in gynecology
Progesterone in gynecologyProgesterone in gynecology
Progesterone in gynecology
 
Uncosummated marriage
Uncosummated marriageUncosummated marriage
Uncosummated marriage
 
Baiju dhea 15.11.12
Baiju dhea 15.11.12Baiju dhea 15.11.12
Baiju dhea 15.11.12
 
Erectile Dysfunction:Evaluation and Management by Dr Shahjada Selim
Erectile Dysfunction:Evaluation and Management by Dr Shahjada SelimErectile Dysfunction:Evaluation and Management by Dr Shahjada Selim
Erectile Dysfunction:Evaluation and Management by Dr Shahjada Selim
 
Role of antioxidant in male infertility
Role of  antioxidant in male infertilityRole of  antioxidant in male infertility
Role of antioxidant in male infertility
 
Letrozole ovulation induction
Letrozole ovulation inductionLetrozole ovulation induction
Letrozole ovulation induction
 
Erectile Dysfunction
Erectile DysfunctionErectile Dysfunction
Erectile Dysfunction
 
Androgens and antiandrogens
Androgens and antiandrogensAndrogens and antiandrogens
Androgens and antiandrogens
 
Uses of of N-acetyl Cysteine in Medicine, obstetrics, gynecology and inferti...
Uses of  of N-acetyl Cysteine in Medicine, obstetrics, gynecology and inferti...Uses of  of N-acetyl Cysteine in Medicine, obstetrics, gynecology and inferti...
Uses of of N-acetyl Cysteine in Medicine, obstetrics, gynecology and inferti...
 

Similar to erectile dysfunction

SHCRC Dr. Ajay Gupta Presentation NCS2010
SHCRC Dr. Ajay Gupta Presentation NCS2010SHCRC Dr. Ajay Gupta Presentation NCS2010
SHCRC Dr. Ajay Gupta Presentation NCS2010shcrc
 
SSRIs Sexual Dysfunction.pptx
SSRIs Sexual Dysfunction.pptxSSRIs Sexual Dysfunction.pptx
SSRIs Sexual Dysfunction.pptxlarry lee
 
Nutrition And Erectile Dysfunction Public
Nutrition And Erectile Dysfunction PublicNutrition And Erectile Dysfunction Public
Nutrition And Erectile Dysfunction PublicRosen Wellness, LLC
 
Erectile Dysfunction.pptx
Erectile Dysfunction.pptxErectile Dysfunction.pptx
Erectile Dysfunction.pptxSoumen Karmakar
 
Aging And Sexual Function
Aging And Sexual FunctionAging And Sexual Function
Aging And Sexual FunctionMamdouh Sabry
 
4. Erectile dysfunction.pptx
4. Erectile dysfunction.pptx4. Erectile dysfunction.pptx
4. Erectile dysfunction.pptxJibrilAliSe
 
Ueda2015 d erectile dysfunction patients_dr.khaled mohy
Ueda2015 d erectile dysfunction patients_dr.khaled mohyUeda2015 d erectile dysfunction patients_dr.khaled mohy
Ueda2015 d erectile dysfunction patients_dr.khaled mohyueda2015
 
Sexuality and sexual dysfunction
Sexuality and sexual dysfunctionSexuality and sexual dysfunction
Sexuality and sexual dysfunctiondrmcbansal
 
Causes of Female Sexual Arousal Disorder
Causes of Female Sexual Arousal DisorderCauses of Female Sexual Arousal Disorder
Causes of Female Sexual Arousal Disordersexologistdoctors
 
Nguyên nhân gây suy giảm ham muốn tình dục ở phụ nữ và cách điều trị | Venus ...
Nguyên nhân gây suy giảm ham muốn tình dục ở phụ nữ và cách điều trị | Venus ...Nguyên nhân gây suy giảm ham muốn tình dục ở phụ nữ và cách điều trị | Venus ...
Nguyên nhân gây suy giảm ham muốn tình dục ở phụ nữ và cách điều trị | Venus ...VENUS
 
Disturbances of piturtary adrenal gonadal axis in hemodialysis pt
Disturbances of piturtary  adrenal gonadal  axis in hemodialysis ptDisturbances of piturtary  adrenal gonadal  axis in hemodialysis pt
Disturbances of piturtary adrenal gonadal axis in hemodialysis ptalaa wafa
 

Similar to erectile dysfunction (20)

SHCRC Dr. Ajay Gupta Presentation NCS2010
SHCRC Dr. Ajay Gupta Presentation NCS2010SHCRC Dr. Ajay Gupta Presentation NCS2010
SHCRC Dr. Ajay Gupta Presentation NCS2010
 
SSRIs Sexual Dysfunction.pptx
SSRIs Sexual Dysfunction.pptxSSRIs Sexual Dysfunction.pptx
SSRIs Sexual Dysfunction.pptx
 
Nutrition And Erectile Dysfunction Public
Nutrition And Erectile Dysfunction PublicNutrition And Erectile Dysfunction Public
Nutrition And Erectile Dysfunction Public
 
Erectile Dysfunction Clinic in San Diego
Erectile Dysfunction Clinic in San DiegoErectile Dysfunction Clinic in San Diego
Erectile Dysfunction Clinic in San Diego
 
Sexual rehabilitation
Sexual rehabilitationSexual rehabilitation
Sexual rehabilitation
 
Sexual dysfunction
Sexual dysfunctionSexual dysfunction
Sexual dysfunction
 
Erectile Dysfunction.pptx
Erectile Dysfunction.pptxErectile Dysfunction.pptx
Erectile Dysfunction.pptx
 
Aging And Sexual Function
Aging And Sexual FunctionAging And Sexual Function
Aging And Sexual Function
 
Ejaculation physiology and pathology
Ejaculation  physiology and pathologyEjaculation  physiology and pathology
Ejaculation physiology and pathology
 
Impotence
ImpotenceImpotence
Impotence
 
Reproductive hormones
Reproductive hormonesReproductive hormones
Reproductive hormones
 
4. Erectile dysfunction.pptx
4. Erectile dysfunction.pptx4. Erectile dysfunction.pptx
4. Erectile dysfunction.pptx
 
Ejaculatory Disorders
Ejaculatory DisordersEjaculatory Disorders
Ejaculatory Disorders
 
Ueda2015 d erectile dysfunction patients_dr.khaled mohy
Ueda2015 d erectile dysfunction patients_dr.khaled mohyUeda2015 d erectile dysfunction patients_dr.khaled mohy
Ueda2015 d erectile dysfunction patients_dr.khaled mohy
 
Sexuality and sexual dysfunction
Sexuality and sexual dysfunctionSexuality and sexual dysfunction
Sexuality and sexual dysfunction
 
Causes of Female Sexual Arousal Disorder
Causes of Female Sexual Arousal DisorderCauses of Female Sexual Arousal Disorder
Causes of Female Sexual Arousal Disorder
 
Erectile dysfunction
Erectile dysfunctionErectile dysfunction
Erectile dysfunction
 
Testosterone
TestosteroneTestosterone
Testosterone
 
Nguyên nhân gây suy giảm ham muốn tình dục ở phụ nữ và cách điều trị | Venus ...
Nguyên nhân gây suy giảm ham muốn tình dục ở phụ nữ và cách điều trị | Venus ...Nguyên nhân gây suy giảm ham muốn tình dục ở phụ nữ và cách điều trị | Venus ...
Nguyên nhân gây suy giảm ham muốn tình dục ở phụ nữ và cách điều trị | Venus ...
 
Disturbances of piturtary adrenal gonadal axis in hemodialysis pt
Disturbances of piturtary  adrenal gonadal  axis in hemodialysis ptDisturbances of piturtary  adrenal gonadal  axis in hemodialysis pt
Disturbances of piturtary adrenal gonadal axis in hemodialysis pt
 

More from somayyeh nasiripour

More from somayyeh nasiripour (12)

Brucellosis
BrucellosisBrucellosis
Brucellosis
 
Urinary tract infection
Urinary tract infectionUrinary tract infection
Urinary tract infection
 
Hypersensitivity to Contrast Media
Hypersensitivity to Contrast MediaHypersensitivity to Contrast Media
Hypersensitivity to Contrast Media
 
Supplements and vitamins
Supplements and vitaminsSupplements and vitamins
Supplements and vitamins
 
Treatment with anticoagulation
Treatment with anticoagulationTreatment with anticoagulation
Treatment with anticoagulation
 
Nitrates
NitratesNitrates
Nitrates
 
Surgical prophylaxis
Surgical prophylaxisSurgical prophylaxis
Surgical prophylaxis
 
Rational use of NSAIDS
Rational use of NSAIDSRational use of NSAIDS
Rational use of NSAIDS
 
ADR chemotherapy.chemotherapy adverse effect
ADR chemotherapy.chemotherapy adverse effect ADR chemotherapy.chemotherapy adverse effect
ADR chemotherapy.chemotherapy adverse effect
 
Hepatitis
HepatitisHepatitis
Hepatitis
 
the role of diet & supplements in cancer
the role of diet & supplements in cancer the role of diet & supplements in cancer
the role of diet & supplements in cancer
 
Urinary tract infection by Somayyeh Nasiripour,Pharm.D,assistant professor
Urinary tract infection by Somayyeh Nasiripour,Pharm.D,assistant professorUrinary tract infection by Somayyeh Nasiripour,Pharm.D,assistant professor
Urinary tract infection by Somayyeh Nasiripour,Pharm.D,assistant professor
 

Recently uploaded

Beyond the EU: DORA and NIS 2 Directive's Global Impact
Beyond the EU: DORA and NIS 2 Directive's Global ImpactBeyond the EU: DORA and NIS 2 Directive's Global Impact
Beyond the EU: DORA and NIS 2 Directive's Global ImpactPECB
 
1029 - Danh muc Sach Giao Khoa 10 . pdf
1029 -  Danh muc Sach Giao Khoa 10 . pdf1029 -  Danh muc Sach Giao Khoa 10 . pdf
1029 - Danh muc Sach Giao Khoa 10 . pdfQucHHunhnh
 
Call Girls in Dwarka Mor Delhi Contact Us 9654467111
Call Girls in Dwarka Mor Delhi Contact Us 9654467111Call Girls in Dwarka Mor Delhi Contact Us 9654467111
Call Girls in Dwarka Mor Delhi Contact Us 9654467111Sapana Sha
 
CARE OF CHILD IN INCUBATOR..........pptx
CARE OF CHILD IN INCUBATOR..........pptxCARE OF CHILD IN INCUBATOR..........pptx
CARE OF CHILD IN INCUBATOR..........pptxGaneshChakor2
 
Grant Readiness 101 TechSoup and Remy Consulting
Grant Readiness 101 TechSoup and Remy ConsultingGrant Readiness 101 TechSoup and Remy Consulting
Grant Readiness 101 TechSoup and Remy ConsultingTechSoup
 
Nutritional Needs Presentation - HLTH 104
Nutritional Needs Presentation - HLTH 104Nutritional Needs Presentation - HLTH 104
Nutritional Needs Presentation - HLTH 104misteraugie
 
Kisan Call Centre - To harness potential of ICT in Agriculture by answer farm...
Kisan Call Centre - To harness potential of ICT in Agriculture by answer farm...Kisan Call Centre - To harness potential of ICT in Agriculture by answer farm...
Kisan Call Centre - To harness potential of ICT in Agriculture by answer farm...Krashi Coaching
 
Mastering the Unannounced Regulatory Inspection
Mastering the Unannounced Regulatory InspectionMastering the Unannounced Regulatory Inspection
Mastering the Unannounced Regulatory InspectionSafetyChain Software
 
Privatization and Disinvestment - Meaning, Objectives, Advantages and Disadva...
Privatization and Disinvestment - Meaning, Objectives, Advantages and Disadva...Privatization and Disinvestment - Meaning, Objectives, Advantages and Disadva...
Privatization and Disinvestment - Meaning, Objectives, Advantages and Disadva...RKavithamani
 
Introduction to AI in Higher Education_draft.pptx
Introduction to AI in Higher Education_draft.pptxIntroduction to AI in Higher Education_draft.pptx
Introduction to AI in Higher Education_draft.pptxpboyjonauth
 
Contemporary philippine arts from the regions_PPT_Module_12 [Autosaved] (1).pptx
Contemporary philippine arts from the regions_PPT_Module_12 [Autosaved] (1).pptxContemporary philippine arts from the regions_PPT_Module_12 [Autosaved] (1).pptx
Contemporary philippine arts from the regions_PPT_Module_12 [Autosaved] (1).pptxRoyAbrique
 
Organic Name Reactions for the students and aspirants of Chemistry12th.pptx
Organic Name Reactions  for the students and aspirants of Chemistry12th.pptxOrganic Name Reactions  for the students and aspirants of Chemistry12th.pptx
Organic Name Reactions for the students and aspirants of Chemistry12th.pptxVS Mahajan Coaching Centre
 
Sanyam Choudhary Chemistry practical.pdf
Sanyam Choudhary Chemistry practical.pdfSanyam Choudhary Chemistry practical.pdf
Sanyam Choudhary Chemistry practical.pdfsanyamsingh5019
 
“Oh GOSH! Reflecting on Hackteria's Collaborative Practices in a Global Do-It...
“Oh GOSH! Reflecting on Hackteria's Collaborative Practices in a Global Do-It...“Oh GOSH! Reflecting on Hackteria's Collaborative Practices in a Global Do-It...
“Oh GOSH! Reflecting on Hackteria's Collaborative Practices in a Global Do-It...Marc Dusseiller Dusjagr
 
POINT- BIOCHEMISTRY SEM 2 ENZYMES UNIT 5.pptx
POINT- BIOCHEMISTRY SEM 2 ENZYMES UNIT 5.pptxPOINT- BIOCHEMISTRY SEM 2 ENZYMES UNIT 5.pptx
POINT- BIOCHEMISTRY SEM 2 ENZYMES UNIT 5.pptxSayali Powar
 
1029-Danh muc Sach Giao Khoa khoi 6.pdf
1029-Danh muc Sach Giao Khoa khoi  6.pdf1029-Danh muc Sach Giao Khoa khoi  6.pdf
1029-Danh muc Sach Giao Khoa khoi 6.pdfQucHHunhnh
 
Paris 2024 Olympic Geographies - an activity
Paris 2024 Olympic Geographies - an activityParis 2024 Olympic Geographies - an activity
Paris 2024 Olympic Geographies - an activityGeoBlogs
 

Recently uploaded (20)

Beyond the EU: DORA and NIS 2 Directive's Global Impact
Beyond the EU: DORA and NIS 2 Directive's Global ImpactBeyond the EU: DORA and NIS 2 Directive's Global Impact
Beyond the EU: DORA and NIS 2 Directive's Global Impact
 
1029 - Danh muc Sach Giao Khoa 10 . pdf
1029 -  Danh muc Sach Giao Khoa 10 . pdf1029 -  Danh muc Sach Giao Khoa 10 . pdf
1029 - Danh muc Sach Giao Khoa 10 . pdf
 
INDIA QUIZ 2024 RLAC DELHI UNIVERSITY.pptx
INDIA QUIZ 2024 RLAC DELHI UNIVERSITY.pptxINDIA QUIZ 2024 RLAC DELHI UNIVERSITY.pptx
INDIA QUIZ 2024 RLAC DELHI UNIVERSITY.pptx
 
Call Girls in Dwarka Mor Delhi Contact Us 9654467111
Call Girls in Dwarka Mor Delhi Contact Us 9654467111Call Girls in Dwarka Mor Delhi Contact Us 9654467111
Call Girls in Dwarka Mor Delhi Contact Us 9654467111
 
CARE OF CHILD IN INCUBATOR..........pptx
CARE OF CHILD IN INCUBATOR..........pptxCARE OF CHILD IN INCUBATOR..........pptx
CARE OF CHILD IN INCUBATOR..........pptx
 
Grant Readiness 101 TechSoup and Remy Consulting
Grant Readiness 101 TechSoup and Remy ConsultingGrant Readiness 101 TechSoup and Remy Consulting
Grant Readiness 101 TechSoup and Remy Consulting
 
Nutritional Needs Presentation - HLTH 104
Nutritional Needs Presentation - HLTH 104Nutritional Needs Presentation - HLTH 104
Nutritional Needs Presentation - HLTH 104
 
Kisan Call Centre - To harness potential of ICT in Agriculture by answer farm...
Kisan Call Centre - To harness potential of ICT in Agriculture by answer farm...Kisan Call Centre - To harness potential of ICT in Agriculture by answer farm...
Kisan Call Centre - To harness potential of ICT in Agriculture by answer farm...
 
Mattingly "AI & Prompt Design: The Basics of Prompt Design"
Mattingly "AI & Prompt Design: The Basics of Prompt Design"Mattingly "AI & Prompt Design: The Basics of Prompt Design"
Mattingly "AI & Prompt Design: The Basics of Prompt Design"
 
Mastering the Unannounced Regulatory Inspection
Mastering the Unannounced Regulatory InspectionMastering the Unannounced Regulatory Inspection
Mastering the Unannounced Regulatory Inspection
 
Código Creativo y Arte de Software | Unidad 1
Código Creativo y Arte de Software | Unidad 1Código Creativo y Arte de Software | Unidad 1
Código Creativo y Arte de Software | Unidad 1
 
Privatization and Disinvestment - Meaning, Objectives, Advantages and Disadva...
Privatization and Disinvestment - Meaning, Objectives, Advantages and Disadva...Privatization and Disinvestment - Meaning, Objectives, Advantages and Disadva...
Privatization and Disinvestment - Meaning, Objectives, Advantages and Disadva...
 
Introduction to AI in Higher Education_draft.pptx
Introduction to AI in Higher Education_draft.pptxIntroduction to AI in Higher Education_draft.pptx
Introduction to AI in Higher Education_draft.pptx
 
Contemporary philippine arts from the regions_PPT_Module_12 [Autosaved] (1).pptx
Contemporary philippine arts from the regions_PPT_Module_12 [Autosaved] (1).pptxContemporary philippine arts from the regions_PPT_Module_12 [Autosaved] (1).pptx
Contemporary philippine arts from the regions_PPT_Module_12 [Autosaved] (1).pptx
 
Organic Name Reactions for the students and aspirants of Chemistry12th.pptx
Organic Name Reactions  for the students and aspirants of Chemistry12th.pptxOrganic Name Reactions  for the students and aspirants of Chemistry12th.pptx
Organic Name Reactions for the students and aspirants of Chemistry12th.pptx
 
Sanyam Choudhary Chemistry practical.pdf
Sanyam Choudhary Chemistry practical.pdfSanyam Choudhary Chemistry practical.pdf
Sanyam Choudhary Chemistry practical.pdf
 
“Oh GOSH! Reflecting on Hackteria's Collaborative Practices in a Global Do-It...
“Oh GOSH! Reflecting on Hackteria's Collaborative Practices in a Global Do-It...“Oh GOSH! Reflecting on Hackteria's Collaborative Practices in a Global Do-It...
“Oh GOSH! Reflecting on Hackteria's Collaborative Practices in a Global Do-It...
 
POINT- BIOCHEMISTRY SEM 2 ENZYMES UNIT 5.pptx
POINT- BIOCHEMISTRY SEM 2 ENZYMES UNIT 5.pptxPOINT- BIOCHEMISTRY SEM 2 ENZYMES UNIT 5.pptx
POINT- BIOCHEMISTRY SEM 2 ENZYMES UNIT 5.pptx
 
1029-Danh muc Sach Giao Khoa khoi 6.pdf
1029-Danh muc Sach Giao Khoa khoi  6.pdf1029-Danh muc Sach Giao Khoa khoi  6.pdf
1029-Danh muc Sach Giao Khoa khoi 6.pdf
 
Paris 2024 Olympic Geographies - an activity
Paris 2024 Olympic Geographies - an activityParis 2024 Olympic Geographies - an activity
Paris 2024 Olympic Geographies - an activity
 

erectile dysfunction

  • 1. Erectile dysfunction (ED) By: Somayyeh Nasiripour,Pharm.D Board of clinical pharmacy Assistant professor at IUMS
  • 2. PHYSIOLOGY OF MALE SEXUAL FUNCTION  Normal male sexual function requires interactions among vascular, neurologic, hormonal, and psychological systems.   The initial obligatory event required for male sexual activity, is a vascular phenomenon,  triggered by neurologic signals and facilitated only in the presence of an appropriate hormonal milieu and psychological mindset.
  • 3. Psychogenic erections  triggered by neural impulses  may originate in response to erotic visual or auditory stimuli or be generated via fantasy.   The centrally perceived sensual input is relayed by neural signals to a spinal cord neural center located at T-11 to L-2 (the thoracolumbar erection center).  From there, neural impulses flow to the pelvic vascular bed, redirecting blood into the corpora cavernosae
  • 4. Reflex erections  created by tactile stimulus to the penis or genital area which activates a reflex arc with sacral roots originating at S-2 to S-4 (the sacral erection center).  Psychogenic erections are more common during man’s early sexually active years, whereas reflex erectile activity dominates during his mature years
  • 5. Nonsexual, nocturnal erections  occurring three to four times nightly, start in early adolescence.  Nocturnal erectile activity may go unnoticed by sleeping men, although most men will be aware of an erection when they arise in the morning.  These early morning erections often fade after urination, creating the incorrect impression that they are a reflex response to a full bladder.  Nocturnal erections occur only during rapid eye movement (REM) sleep  depressed men rarely experience REM sleep and do not have nocturnal or early morning erections.  Nocturnal erections persist throughout life, although, for as yet unexplained reasons, nocturnal erectile activity is not as tightly coupled to REM sleep in older men
  • 6.
  • 7.
  • 8. Role of blood flow & NO  Normal erections require blood to flow from the hypogastric arterial system to the paired corpora cavernosae  As blood flow accelerates, the pressure within the intracavernosal spaces increases dramatically, preventing penile venous outflow from emissary veins.  This combination of increased intracavernosal blood flow and reduced venous outflow allows a man maintain a firm erection.  High levels of intrapenile nitric oxide act as a local neurotransmitter to facilitate the relaxation of intracavernosal trabeculae, thereby maximizing blood flow and penile engorgement  NO formed under the influence of the enzyme NO synthase, which, in conjunction with NADPH and oxygen, transforms the substrate amino acid arginine to citrulline and nitric oxide.
  • 9.
  • 10.  The absolute prerequisites for penile erectile activity are an adequate arterial inflow to provide a constant source of intracavernosal oxygen and sufficient nitric oxide synthase to generate nitric oxide.  Nitric oxide acts by promoting the generation of cyclic GMP.  (loss of erection) occurs when nitric oxide-induced vasodilation disappears because of metabolism of cyclic GMP, which is primarily mediated by intracavernosal type 5 cyclic GMP
  • 11.  The role of nitric oxide may have important therapeutic implications for patients with erectile dysfunction (ED).   Low intracavernosal nitric oxide synthase levels are found in cigarette smokers and patients with diabetes and testosterone deficiency, which may explain why these factors are associated with a high frequency of ED.  sildenafil , vardenafil and tadalafil , are (PDE-5) inhibitors.
  • 12.  Interference with oxygen delivery or nitric oxide synthesis can prevent intracavernosal blood pressure from rising to a level sufficient to impede venous outflow, leading to an inability to acquire or sustain a rigid erection.  Examples include decreased blood flow and inadequate intracavernosal oxygen levels when atherosclerosis involves the hypogastric artery or other feeder vessels  diabetes mellitus, which is associated with suboptimal nitric oxide synthase activity
  • 13. Does ED related to hormonal deficiency
  • 14. Hormonal influences  Testosterone plays an integral role in normal male sexual function.  The onset of adolescent nocturnal erections coincides with the pulsatile release of (GnRH) (LH) activation of Leydig cell testosterone  Testosterone deficiency results in impotence in experimental animals and men,  sexual potency returns when testosterone levels are normalized
  • 15. • Testosterone acts through psychogenic channels to enhance libido. • Testosterone is necessary for maintenance of intrapenile nitric oxide synthase levels
  • 16. Can Normal sexual activity be related to aging ?????
  • 17. Normal sexual activity with age  Sexual activity is affected by age, health status, and gender.  In a population-based survey, men were more likely than women to be sexually active and report a good quality sex life .  Gender differences increased with age and were most noticeable in the 75 to 85 year old group: 39 percent of men versus 17 percent of women were sexually active.  Men and women in good health were more likely to be sexually active compared to those in fair or poor health.  Men lost more years of sexually active life as a result of poor health than women.  Although men may remain sexually active, there are a number of age-associated changes in sexual function in men including delay in erection, diminished intensity and duration of orgasm, and decreased force of seminal emission  nine-year longitudinal follow-up study confirmed the age-associated declines in most domains of sexual function: sexual intercourse, erection frequency, sexual desire, satisfaction with sex, and orgasm
  • 20.  prevalence of reduced libido 5 to 15 % in men.  It increases with age  frequently accompanies other sexual disorders.  Men with ED may experience loss of libido as a secondary consequence of ED. This usually is ascertained from a detailed sexual history, including the chronology of the disorder.  However, most patients who complain of erectile dysfunction (ED) do not complain of reduced libido or sexual desire.
  • 21. causes of low libido include • Medications (SSRIs, anti-androgens, 5-alpha reductase inhibitors, opioid analgesics) • Alcoholism • Depression • Fatigue • Hypoactive sexual disorder • Recreational drugs • Relationship problems • Other sexual dysfunction (fear of humiliation) • Sexual aversion disorder • Systemic illness • Testosterone deficiency
  • 22.  Most of these conditions are potentially treatable, so it is important to take a good medical history, perform a careful examination, and obtain relevant laboratory studies to determine if any of them are present.
  • 23. Sexual dysfunction associated with selective serotonin reuptake inhibitor (SSRI) antidepressants
  • 24.  reported to reduce libido in women and men, to cause anorgasmia in women, and to increase ejaculation latency in men  Women are treated with SSRI medications more frequently than men (in a ratio of 2:1) and are therefore more likely to be affected by SSRI-related sexual dysfunction  Estimates of the incidence of sexual dysfunction with fluoxetine have been higher than initially thought and may be 50 percent or more  The incidence of adverse effects on sexual function with other SSRIs is not clear, although all SSRIs have the propensity to cause sexual side effects  Of note, many other antidepressants (MAOIs), TCA , venlafaxine and duloxetine have been associated with sexual side effects, although this phenomenon has been less well studied than with SSRIs.
  • 25. MANAGEMENT OF SEXUAL DYSFUNCTION •Decrease the dose •Switch to another SSRI •Switch to a non- SSRI •Use a second drug to offset the adverse effects •Drug holidays
  • 26. Switch to a non-SSRI  Bupropion , nefazodone , and mirtazapine appear to have no or only a limited effect on sexual function  Switching to duloxetine , released in the United States in 2004, may also be an option.  rates of sexual side effects in placebo-controlled trials of duloxetine appear to be relatively low, particularly in women
  • 27. Use of a second drug Phosphodiesterase-5 (PDE-5) inhibitors Bupropion Buspirone cyproheptadine stimulants amantadine yohimbine
  • 28. (PDE-5) inhibitors  Several studies indicate that the phosphodiesterase-5 inhibitor sildenafil helps relieve sexual dysfunction caused by antidepressants  Improvement of sexual functioning (including erectile function, arousal, ejaculation, orgasm, and overall satisfaction) occurred in more patients with sildenafil than placebo (55 versus 4 %.  sildenafil is not helpful in treating decreased libido associated with SSRIs.  Other phosphodiesterase-5 inhibitors are probably useful as well  A randomized trial of sildenafil 50 or 100 mg in 98 women with major depression in remission (analogous to the study in men described above found that sildenafil for eight weeks, compared to placebo, significantly improved scores on the Clinical Global Impression sexual function scale
  • 29. Bupropion  (150 mg twice daily) demonstrated improvement in sexual desire at four weeks, compared to placebo,  in those who could tolerate the higher dose; there was a 25 % drop out, due to bupropion side effects (dry mouth, headache)  A systematic review discussed a third randomized study which also demonstrated improvement in desire, though not orgasm, in women treated with an SSRI and bupropion 150 mg compared to SSRI plus placebo
  • 30. Buspirone  STUDEIS have different result  effect of buspirone is reduced with time  We have not found buspirone to be particularly helpful in treating SSRI-associated sexual dysfunction. Drug holidays
  • 31. Drug holidays  Drug holidays have not been well studied in patients with SSRI-induced sexual dysfunction.  An open-label nonrandomized study of weekend drug holidays found no benefit for patients taking fluoxetine  inconsistent results for patients taking paroxetine and sertraline
  • 32. Erectile dysfunction consistent or recurrent inability to acquire or sustain an erection of sufficient rigidity and duration for sexual intercourse
  • 33. Prevalence  The frequency of sexual activity decreases with age in both men and women, and sexual problems become more common  In men, the most common sexual problem is erectile dysfunction (ED)  The ages ranged from 20 to 75 years, and the overall prevalence of ED was 16 percent. It was 8 percent in men 20 to 30 years of age and 37 percent in men 70 to 75 years of age.
  • 34. Risk factors Low exersise Obesity Low sexual intercourse smoking Dyslipidemia diabetes mellitus, hypertension, obesity, cardiovascular disease medication use Psychosocial factors Neurologic causes (stroke, spinal cord or back injury, multiple sclerosis, or dementia.)
  • 35. Association with cardiovascular disease  Erectile dysfunction and cardiovascular disease share many risk factors, and their pathophysiology is mediated through endothelial dysfunction  Cardiovascular disease and its risk factors increase the risk for later  erectile dysfunction may be an early warning sign of future cardiovascular events  men with ED without an obvious cause (eg, pelvic trauma) and no symptoms of coronary or other vascular disease should be screened for cardiovascular disease and its associated risk factors prior to initiating therapy for their sexual dysfunction, since there are potential cardiac risks associated with sexual activity in patients with heart disease
  • 36. Drugs(25 % of cases of ED) Antidepressants - Most antidepressants are associated with ED, but in particular, the selective serotonin reuptake inhibitors (SSRIs) Spironolactone Sympathetic blockers such as clonidine , guanethidine, or methyldopa Thiazide diuretics Ketoconazole Cimetidine , but apparently not ranitidine or famotidine
  • 37.
  • 38. It had been thought that beta blockers are an important cause of ED, but a systematic review of randomized, controlled trials found only a small increased risk of sexual dysfunction with beta blocker therapy Common over-the-counter products contain chemicals like nicotine and alcohol that have a disruptive effect on male sexual function Although some recreational drugs, such as cocaine and heroin, can initially stimulate libido and sexual arousal, they ultimately exert a negative impact on the ability to acquire and sustain erectile function
  • 39. association of ED with bicycling prolonged pressure on the pudendal and cavernosal nerves or compromises blood flow to the cavernosal artery can result in penile numbness and impotence. The penile numbness has been attributed to the pressure on the perineal nerves ED is thought to be due to a decrease in oxygen pressure in the pudendal arteries
  • 40. Endocrine disorders  Testosterone deficiency affects peripheral mechanisms that are responsible for penile erections  Testosterone deficient men are still capable of exhibiting some erectile activity during nocturnal However, the penile swelling in this setting usually is not of sufficient rigidity to permit vaginal penetration.  This defect is corrected after normalization of testosterone levels, probably due to restoration of intrapenile nitric oxide synthase levels.  In a study of 1162 men, serum testosterone levels <225 ng/dL associated with an increased frequency of sexual dysfunction  he effects of testosterone on libido are more consistent than on erectile function
  • 41.  Testosterone replacement can restore nocturnal erections in hypogonadal men but the effects are greater when testosterone plus a PDE-5 inhibitor are administered  Other disruptions in hormone secretion, including hyperprolactinemia, hyperthyroidism, and hypothyroidism are commonly associated with ED  recognition that 1/3 of men with type 2 DM have subnormal testosterone concentrations suggests that this hormone deficiency, and not just diabetic vasculopathy/neuropathy, may play a role in the ED so commonly seen in men with diabetes
  • 42. Treatment of male sexual dysfunction
  • 43. GENERAL PRINCIPLES  first step in the treatment of male sexual dysfunction is to determine the etiology. Certain clues in the history and appropriate laboratory testing can usually establish the primary cause
  • 44. Clinical clues to causes of male sexual dysfunctio • Psychogenic • Genitourinary trauma - eg, radical prostatectomy Rapid onset • Anxiety • Vascular steal Nonsustained erection • Depression • Drug-induced Depression or use of certain drugs Complete loss of nocturnal erections Vascular disease Neurologic disease
  • 45. Therapy of men with erectile dysfunction is aimed at restoration of the two vital sexual functions: the capacity to acquire and sustain penile erections; reactivation of libido. Optimal treatment varies with the cause of the erectile dysfunction Phosphodiesterase-5 (PDE-5) inhibitors, penile self-injection programs with vasoactive drugs, vacuum erection devices, or penile prostheses allow many men with vasculogenic, neurogenic, or psychogenic erectile dysfunction to acquire and maintain erections.
  • 46. For first-line therapy, we recommend the phosphodiesterase inhibitors because of their efficacy, ease of use, and favorable side effect profile. Sildenafil , vardenafil , and tadalafil appear to be equally effective, but tadalafil has a longer duration of action. (See 'Pharmacotherapy' below and 'Choice of drug' below.) Phosphodiesterase inhibitors are contraindicated in men taking nitrates . For second-line therapy, we recommend consideration of penile self-injectable drugs, intraurethral alprostadil , and vacuum devices. (See 'Penile self-injection' below and 'Vacuum-assisted erection devices' below.) Surgical implantation of a penile prosthesis should be reserved for men who cannot use or who have not responded to first- and second-line therapies. (See 'Penile prostheses' below.) For men with sexual dysfunction and low serum testosterone levels, ie, hypogonadism, testosterone replacement therapy should be the initial treatment. (See "Testosterone treatment of male hypogonadism" .) Psychotherapy alone or in combination with psychoactive drugs may be helpful in men whose erectile dysfunction is caused by depression or anxiety. In addition, alpha adrenergic blocking agents such as yohimbine hydrochloride have enjoyed some popularity as a remedy for men with psychogenic erectile dysfunction. (See "Overview of male sexual dysfunction" .) In obese men with erectile dysfunction, weight loss and increased physical activity are associated with an improvement in erectile function in about one third of patients
  • 47. Lifestyle modifications such as diet and exercise have improved ED in studies of men with hypertension, diabetes, or the metabolic syndrome A meta-analysis of four randomized trials of lifestyle modification interventions in 597 men with ED demonstrated statistically significant improvement in sexual function Statins improved the response to sildenafil in two studies of men with dyslipidemia and ED
  • 49. Phosphodiesterase 5 inhibitors  treatment options now include three phosphodiesterase-5 (PDE-5) inhibitors: sildenafil , vardenafil , and tadalafil .  All act to increase intracavernosal cyclic GMP levels,  and each one has been proven to be effective in restoring erectile function, allowing for satisfactory sexual intercourse in many men with erectile dysfunction.  However, PDE-5 inhibitors will not work without sufficient environmental and psychological cues that result in sufficient sexual arousal and stimulation to initiate the physiological changes in the penis.
  • 51. Sildenafil  Sildenafil (Viagra) was the first orally administered treatment of proven efficacy for erectile dysfunction. Taken one hour before planned sexual intercourse, it is effective for a wide range of disorders causing erectile dysfunction. Nevertheless, it should not be prescribed before performing a systematic assessment of the cause of erectile dysfunction, including a history and physical examination and when appropriate, measurement of serum testosterone, prolactin, and TSH.  rationale for the use of sildenafil and other PDE-5 inhibitors is based upon the role of nitric oxide-induced vasodilation (which is mediated by cyclic GMP) in initiating and maintaining an erection.  Detumescence is associated with catabolism of cyclic GMP by type 5 phosphodiesterase.  PDE-5 inhibitors act by blocking the latter enzyme and as a result increase both the number and duration of erections in men with erectile dysfunction
  • 52. Efficacy  In a dose-escalation study of 25 to 100 mg of sildenafil in 532 men with organic (70 percent), psychogenic (11 percent), or mixed erectile dysfunction (18 percent), 69 % of all attempts at sexual intercourse were successful in the men who took sildenafil compared to 22 percent of those who took placebo  All do not respond to the drug  56 % of diabetic men have improved erections with sildenafil  In men with prostate cancer who have undergone prostatectomy or radiation therapy, sildenafil appears to be effective in approximately 50 % of patients
  • 54. addition of exogenous testosterone to sildenafil therapy may be useful in men with serum total testosterone concentrations <400 ng/dL who do not respond to sildenafil alone. This was illustrated in a study of 75 such patients who were randomly assigned to receive daily testosterone gel (5 gm/day) or placebo in addition to sildenafil A greater improvement in erectile function (as measured by the IIEF) was observed in the testosterone-sildenafil group when compared with placebo. Although serum total and free testosterone concentrations increased in the testosterone-sildenafil group, no correlation was seen between testosterone levels and erectile function. The clinical implications of these findings are unclear, as the lower limit of normal for most total testosterone assays is <280 to 300 ng/dL. Therefore, many of the men in this study were not truly hypogonadal.
  • 55. Dose  For maximum effectiveness, sildenafil should be taken orally about one hour before a planned sexual encounter.  The initial dose should be 50 mg,  it should be reduced to 25 mg if side effects occur.  If, well tolerated but the erectile response is not fully satisfactory, the dose can be increased to 100 mg.  The duration of action is approximately four hours
  • 56. Cardiovascular effects Sildenafil is a vasodilator that lowers the blood pressure by about 8 mmHg; this change typically produces no symptoms The combination of sildenafil and nitrates can lead to severe hypotension (eg, more than 50/25 mmHg) and syncope. As a result, sildenafil is contraindicated in patients taking nitrates of any form, regularly or intermittently. If a man who has taken sildenafil has an acute ischemic syndrome, nitrates should not be prescribed within 24 hours (or longer in patients with renal or hepatic dysfunction). Sildenafil has been associated with myocardial infarction and sudden death. However, since sexual activity is also associated with these complications, case reports of MI in association with sildenafil may have been unrelated to the drug. Consistent with this hypothesis are the results of extensive postmarketing surveillance, as well as a meta- analysis, which have failed to demonstrate a significant association between sildenafil and cardiac events Sildenafil does not appear to have adverse effects on coronary hemodynamics, even in men with either stable or severe coronary disease These data support the positions of the Second Princeton Consensus Panel and a consensus statement from the American College of Cardiology/American Heart Association that a PDE-5 inhibitor is safe for men with stable coronary artery disease who are not taking nitrates The vasodilator properties of sildenafil may have an adverse effect in some patients with a hypertrophic cardiomyopathy; the decrease in preload and afterload can increase the outflow obstruction, culminating in an unstable hemodynamic state
  • 57. Interactions  concurrent use (regularly/intermittently) of organic nitrates in any form (eg, nitroglycerin, isosorbide dinitrate) is X interaction and should be avoided  Alpha adrenergic antagonists, which are commonly used for the treatment of benign prostatic hyperplasia, may cause symptomatic hypotension when taken in combination with PDE-5 inhibitors  Tamsulosin is the least likely to cause orthostatic hypotension  Current labeling for all three PDE-5 inhibitors recommends that patients should be stable on alpha blocker therapy prior to initiating a PDE-5 inhibitor (which should then be started at the lowest recommended dose).  Conversely, in patients already taking a PDE-5 inhibitor, alpha blocker therapy should be initiated at the lowest dose.  Sildenafil should also be avoided in patients taking drugs that can prolong the half-life of sildenafil by blocking CYP3A4 (including erythromycin , ketoconazole , protease inhibitors, and grapefruit juice); if used, the potential adverse effects should be stressed with the patient and sildenafil should be started at a dose of 25 mg rather than the 50 mg dose.
  • 58. Visual effects  Although sildenafil acts by inhibiting type 5 phosphodiesterase, it can affect the retina via weak inhibition of type 6 phosphodiesterase, which is required for the transformation of light into electrical signals.  In a brief report of five normal men, a single 100 mg dose of sildenafil caused changes in the electroretinogram one hour later that were completely reversible at six hours .There were no changes in visual acuity, visual fields, or visual-evoked potentials.  Sildenafil also causes blue vision in approximately 3 percent of men. This effect lasts two to three hours and disappears spontaneously.  Rare cases (23 cases since 1998) of nonarteritic anterior ischemic optic neuropathy (NAION) have been reported in men taking sildenafil and tadalafil  Most of these cases occurred in men with underlying autonomic or vascular risk factors.  NAION shares a number of risk factors with erectile dysfunction: age over 50, hypertension, dyslipidemia, and diabetes. Therefore, it is unclear whether NAION in these patients is due to their underlying vascular risk factors, anatomic defects, or use of sildenafil. However, the United States Food and Drug Administration has ordered that warnings be added to labeling for sildenafil, tadalafil, and vardenafil to reflect this potential risk
  • 59.  The long-term effects of sildenafil on visual function are unknown.  At this time, monitoring of visual function does not appear to be necessary in the average man without retinal disease  Although there are no clinical data on the safety and efficacy of sildenafil in men with retinitis pigmentosa, the manufacturer recommends caution in these patients.
  • 60. Hearing loss  Sildenafil , vardenafil , and tadalafil use have been associated with rare reports (n = 29) of sudden hearing loss  Although no causal relationship has been demonstrated, the FDA has approved labeling changes for all three drugs to include this potential risk.  The hearing loss was temporary in approximately one-third of patients, and ongoing (or unknown) in the remaining two-thirds at the time of the FDA report.
  • 61. Other side effects  Side effects associated with sildenafil are related to its vasodilatory properties and are similar to those induced by nitrates.  These include headache, lightheadedness, dizziness, flushing, distorted vision, and, in some cases, syncope.
  • 62. Recreational use  Because sildenafil treatment is associated with a marked reduction in the post-ejaculatory refractory time, men are capable of having a second erection in a shorter time frame than was possible without this therapy.  As a result, recreational use of sildenafil is common.  ,there are case reports of stroke in men taking high dose sildenafil
  • 64.  Vardenafil shares a similar structure, onset and duration of action and side-effect profile with sildenafil ,  available as a 10 and 20 mg dose  Although there are no direct comparison studies, vardenafil appears to be as effective as sildenafil  The duration of action, like sildenafil, is approximately four hours
  • 65.  High-fat, but not moderate-fat meals, may lower vardenafil 's peak serum concentration by approximately 18 percent, and delay its absorption by one hour  A slight prolongation of the QT interval may occur, but this is not thought to be clinically important  However, vardenafil should not be used in men with congenital QT prolongation or in those taking antiarrhythmics drugs such as quinidine , procainamide , amiodarone , or sotalol .
  • 66. Side effects  similar to those seen with sildenafil , and include headache, flushing, and rhinitis, in 13, 10, and 10 percent, respectively  In one trial, side effects decreased over time  Changes in color vision (blue vision) have not been reported thus far
  • 68.  has a different chemical structure than sildenafil and vardenafil  Although there are no direct comparison studies, tadalafil also appears to be as effective as sildenafil but has a longer duration of action  In addition, it does not cause blue vision (as described below),  but otherwise shares a similar side-effect profile with the other two phosphodiesterase inhibitors.
  • 69. Dose:  The recommended starting dose for as-needed use is 10 mg, increasing to 20 mg if necessary  Lower doses of tadalafil (2.5 mg, 5 mg) are available for once daily administration .this approach appears to be as effective as taking higher doses on an as-needed basis  Food does not interfere with its absorption [ 56 ].  Current FDA labeling includes warnings about daily tadalafil use and drug interactions (the once- daily dose should not exceed 2.5 mg in patients receiving CYP3A4 inhibitors such as ritonavir , ketoconazole , and itraconazole ).  Alcohol should be avoided  caution is advised in patients with hepatic dysfunction.  Daily dose tadalafil should not be used in patients with severe renal insufficiency In an integrated analysis of randomized trials in 1112 men with erectile dysfunction receiving 2.5, 5, 10, or 20 mg of tadalafil or placebo, 75 percent of intercourse attempts were successful in the tadalafil group compared with 32 percent with placebo In a second study in 348 men, 59 percent of intercourse attempts were successful at 36 hours with tadalafil (20 mg) compared with 28 percent with placebo [ 51 ].
  • 70. Side effects  Side effects are similar to those seen with sildenafil and vardenafil with headache, dyspepsia, flushing, and rhinitis occurring in 8 to 14, 5 to 10, 4 to 6, and 5 percent, respectively  The duration of these side effects is not known.  In one study, back pain occurred in 6 percent of patients taking the 20 mg dose .In general, the pain occurred 12 to 24 hours after dosing, was mild, and resolved within 48 hours.  Visual side effects have not been described.  A possible association with sudden hearing loss is described above.
  • 71. Drug interactions  Like sildenafil and vardenafil , tadalafil is contraindicated in men taking concurrent nitrates.  In spite of its longer half life, the time course of nitrate interaction with tadalafil does not appear to be prolonged  However, it has been suggested that nitrates should be avoided for at least 48 hours after the last tadalafil dose  Other issues related to sexual activity in men with coronary heart disease are similar to those with sildenafil.  Alpha adrenergic antagonists, which are commonly used for the treatment of benign prostatic hyperplasia, may cause symptomatic hypotension when taken in combination with PDE-5 inhibitors  Tamsulosin is the least likely to cause orthostatic hypotension  Current labeling for all three PDE-5 inhibitors recommends that patients should be stable on alpha blocker therapy prior to initiating a PDE-5 inhibitor (which should then be started at the lowest recommended dose). Conversely, in patients already taking a PDE-5 inhibitor, alpha blocker therapy should be initiated at the lowest dose.  Excessive alcohol intake (five or more drinks) in combination with tadalafil administration may potentiate the hypotensive effect of tadalafil  Use of potent CYP3A4 inhibitors should be avoided as described above for sildenafil
  • 72. Vision  Rare case reports of nonarteritic anterior ischemic optic neuropathy have been described with the PDE-5 inhibitors, prompting the United States Food and Drug Administration to require revised labeling to reflect this potential risk  The PDE-5 inhibitor in sildenafil cross-reacts with the retinal PDE-6 and accounts for the transient "blue vision" side effect reported in a small percentage of sildenafil-treated men.  Tadalafil does not appear to disrupt PDE-6 so blue vision has not been reported with this medication.  Daily use of either tadalafil or sildenafil does not appear to be associated with retinal toxicity, as illustrated by a trial of 143 men who were randomly assigned to receive sildenafil 50 mg, tadalafil 5 mg, or placebo .After six months of therapy, there were no abnormalities in visual function or electroretinograph recordings suggestive of drug toxicity in the treatment groups compared to placebo.
  • 73. Reproductive effects  Tadalafil does show some cross reactivity with PDE-11, an enzyme that is present in the testes and pituitary, but the physiologic significance of this enzyme is unclear.  Tadalafil does not appear to disrupt any aspect of normal pituitary or testicular function  This was illustrated in a study of 421 men over age 45 who were randomly assigned to receive placebo (n = 101) versus tadalafil 10 mg (n = 103), or placebo (n = 106) versus tadalafil 20 mg (n = 111) daily for six months.  During this interval, when compared to placebo, six months of daily administration of tadalafil caused no change in sperm concentration, sperm count, motility or morphology and did not alter serum LH, FSH or testosterone concentrations.
  • 75.  All PDE-5 inhibitors allow men to have erections after appropriate sexual stimulation but differ in the onset of action as well as duration of effectiveness.  Tadalafil differs in two ways. Its absorption is less affected by high fat meals and alcohol and it has a longer duration of action.  Sildenafil has the longest safety record of the three drugs.  With sildenafil and vardenafil men are advised that maximum effectiveness is achieved by taking the tablet on an empty stomach (high fat meals and alcohol appear to delay the absorption of both drugs) and then wait at least an hour before attempting sexual intercourse.  Tadalafil can be taken without regard to meals.  The period of effectiveness for a single dose of sildenafil or vardenafil is four hours. For men with mild to moderate ED, the efficacy of sildenafil may persist for 8 to 12 hours [  Men with erectile dysfunction may have an erection in response to sexual stimulation for up to 36 hours after a single tadalafil dose
  • 76.  In one international multi-center double-blind crossover study comparing patient preference for sildenafil 50 mg or tadalafil 20 mg, 215 men (mostly sildenafil naïve men) with erectile dysfunction age 65 and younger were randomized to receive each medication for four weeks with an intervening one week washout period before switching to the other medication .The following results were seen:  Men reported a similar number of sexual intercourse episodes: 2334 with tadalafil and 2233 with sildenafil .  At the end of the study, 66.3 percent of men expressed a preference for tadalafil and 33.7 percent for sildenafil as a treatment for their erectile dysfunction.  The interval between dosing and sexual intercourse differed. On average sildenafil -treated men had sex 2.2 hours after dosing well within the 4 hour window of opportunity stipulated on the label whereas tadalafil -treated men were able to maintain efficacy but were able to delay sexual intercourse for 5.5 hours after dosing.
  • 78.  Currently available PDE-5 inhibitors result in similarly high rates of successful sexual intercourse (68 to 69 percent compared to 33 to 35 percent for placebo), and similar side effect profiles  Therefore, a Clinical Practice Guideline from The American College of Physicians recommends that the choice of PDE-5 inhibitor be based upon on the patient's preferences, including cost, ease of use, and adverse effects
  • 80.  The FDA has issued a warning to consumers not to purchase or consume dietary supplements that claim to increase sexual stamina, confidence, and performance  Current studies suggest that one-third to one-half of supplements claiming to be “natural” products for sexual enhancement contain synthetic chemicals, most commonly, PDE-5 inhibitors or analogs of PDE- 5 inhibitors  The concern is that patients who take nitrates for cardiovascular disease may experience a drastic lowering of blood pressure if these supplements are consumed.
  • 82.  All three PDE-5 inhibitors: sildenafil , vardenafil and tadalafil work to sustain levels of cyclic GMP within the penile corpora cavernosae to allow men with erectile dysfunction to achieve erections in response to appropriate sexual stimuli.  Duration of action separates one PDE-5 inhibitor from another. Sildenafil and vardenafil are effective as early as 30 minutes and up to four hours after dosing  whereas tadalafil is effective as early as 16 minutes after and up to 36 hours after dosing.  Sildenafil and vardenafil must be taken on an empty stomach, while tadalafil can be taken without regard to food.  An important factor in the success of PDE-5 inhibitor therapy is instruction and counseling on proper use and administration.  Sildenafil , vardenafil , and tadalafil result in similarly high rates of successful sexual intercourse  Side effect profiles are similar; only rarely do men with erectile dysfunction discontinue treatment because of side effects. Current practice guidelines recommend that the choice of PDE-5 inhibitor should be based upon on the patient's preferences, including cost, ease of use, and adverse effects
  • 83. Penile self-injection Intrapenile injection therapy with alprostadil (prostaglandin E1, Caverject), papaverine , or alprostadil with papaverine and phentolamine (Tri-Mix) have all been used for purposes of inducing erection.
  • 84.  The sympathetic nervous system normally maintains the penis in a flaccid or non-erect state.  All of these vasoactive drugs, when injected into the corpora cavernosae, inhibit or override sympathetic inhibition to encourage relaxation of the smooth muscle trabeculae within the penile erectile bodies.  . The combination of accelerated arterial inflow and impeded venous outflow from the corpora cavernosae creates an erection  Considerable education is required for men to become facile with this form of therapy.  Under the guidance of urologists men are trained in sterile methods and the proper technique for inserting an insulin syringe with a 26 gauge needle through the shaft of the penis and injecting the vasoactive agent into one corporeal body  The cross circulation of the penile corpora allows medication injected into one penile corporeal body to diffuse over to the contralateral side so that a full, firm erection can be expected within a few minutes after intrapenile installation of the drug
  • 85.
  • 86.  In the early days of penile self-injection therapy, papaverine proved to be more reliable than phentolamine in producing an erection.  Papaverine is a parenteral vasodilator with marginal efficacy in peripheral vascular disease,  phentolamine is an alpha-adrenergic blocker used for hypertensive crises in pheochromocytoma.
  • 87. Alprostadil  originally introduced to maintain the patency of the ductus arteriosus before definitive cardiac surgery could be undertaken.  When it became apparent that it was effective in a penile self-injection program, the pharmaceutical company making alprostadil was granted approval by FDA to market alprostadil under the name Caverject as a treatment for erectile dysfunction.  alprostadil and papaverine remain popular as monotherapy, and all three drugs can be given together.
  • 88.  The efficacy and safety of a program of at-home intrapenile alprostadil injections were then investigated in a six-month study  683 men with vasculogenic, neurogenic, psychogenic, and mixed causes of erectile dysfunction entered the trial and 471 (69 percent) completed the trial.  The reasons for drop out included penile pain, lack of efficacy, loss to follow-up, dislike of self injection, and other problems.  satisfactory by 87 percent of the men completing the trial and 86 percent of their partners.
  • 89. Side effects penile pain, occurring in 50 percent. Pain was the side effect most often cited by men who discontinued therapy. Priapism Papaverine injected into the corpora cavernosae may escape into the general circulation and can be hepatotoxic; abnormal liver function tests have been reported in men using this drug for penile self-injection In contrast, alprostadil is metabolized within the corpora and does not adversely affect the liver.
  • 90. Priapism  Priapism, or a prolonged erection lasting more than four to six hours, is a medical emergency often requiring immediate urologic attention to evacuate blood clogged within the corpora cavernosae [ 75 ]. Prolonged erections occur in 6 percent of men who use intrapenile alprostadil and about 11 percent of those who use intrapenile papaverine .  Lower doses should be used in men with neuropathic erectile dysfunction (due to spinal cord injury or multiple sclerosis), due to their risk of priapism.
  • 91. One study evaluated the effects of prolonged priapism  Most priapism that lasted 36 hours could be treated successfully by puncture and alpha-adrenergic drugs without any fibrosis of the corpora cavernosa.  After 48 hours, glandulocavernosal shunts were required to achieve detumescence. All the men developed fibrosis of the corpora cavernosa and all but one were unable to continue with the injections of vasoactive drugs.
  • 93.  Intraurethral administration of alprostadil (MUSE) provides a less invasive alternative to intrapenile injection.  The efficacy of intraurethral alprostadil was evaluated in a double-blind, placebo-controlled trial in 1511 men with chronic erectile dysfunction from a variety of organic causes  2/3 of these men responded to intraurethral alprostadil with an erection sufficient for intercourse;  these men were then randomly assigned to therapy with either alprostadil or placebo. Successful intercourse on at least one occasion was much more likely with alprostadil (65 versus 19 percent with placebo).  Among the men who responded to alprostadil, 7 of 10 applications were followed by successful intercourse. After insertion of the alprostadil into the urethra, the penis should be massaged for up to one minute to ensure equal distribution in the corpora cavernosae.
  • 94. Side effects  systemic effects were uncommon,  complications such as priapism and penile fibrosis were less common than after alprostadil given by penile injection.  The 19 percent response rate in the placebo group suggests that psychogenic factors were responsible for the sexual dysfunction, since placebo injections do not induce erections in solely organic causes of impotence.
  • 95.
  • 97.  increased arterial inflow and occlusive rings to discourage venous egress from the penile corpora cavernosae  A certain amount of mechanical dexterity is required to use these devices effectively, but once men become comfortable with using the vacuum and restraining rings they can create an erection sufficient for vaginal penetration and sexual intercourse.  They cannot, however, ejaculate externally because the occlusive rings that prevent venous drainage also compress the penile urethra sufficiently to prevent seminal fluid from reaching and traversing the urethral meatus.  A number of devices are available for purchase over the counter.  Vacuum devices successfully create erections in as many as 67 percent of patients.  Satisfaction with vacuum-assisted erections has varied between 25 and 49 percent.
  • 98.
  • 99.
  • 101.  This form of therapy remains a viable option for those men who do not respond to sildenafil and find penile injection or vacuum erection therapy distasteful  The placement of penile implants requires surgery  Side effects include those related to the anesthesia, local wound infections, and mechanical failure necessitating surgical removal and reimplantation of a new functioning prosthesis.
  • 102.
  • 104.  blocking presynaptic alpha-2-adrenergic receptors, yohimbine increases cholinergic and decreases adrenergic tone, changes that should theoretically be effective in men with psychogenic erectile dysfunction  Optimal results are achieved when yohimbine use is restricted to men with psychogenic erectile dysfunction  Treatment is continued only in those who respond and also tolerate the drug's side effects, which include dizziness, flushing, nausea, and headache.
  • 106. Testosterone should be administered only to a man who is hypogonadal , as evidenced by clinical symptoms and signs consistent with androgen deficiency and a distinctly subnormal serum testosterone concentration. In comparison, increasing the serum testosterone concentration in a man who has symptoms suggestive of hypogonadism but whose testosterone concentration is already normal will not relieve those symptoms. (See "Clinical features and diagnosis of male hypogonadism" .) Testosterone can be replaced satisfactorily whether the testosterone deficiency is due to primary or secondary hypogonadism. The principal goal of testosterone therapy is to restore the serum testosterone concentration to the normal range The role of testosterone replacement to treat the decline in serum testosterone concentration that occurs with increasing frequency above age 60 in the absence of identifiable pituitary or hypothalamic disease is uncertain